LSAT and Law School Admissions Forum

Get expert LSAT preparation and law school admissions advice from PowerScore Test Preparation.

User avatar
 Dave Killoran
PowerScore Staff
  • PowerScore Staff
  • Posts: 5850
  • Joined: Mar 25, 2011
|
#88300
Complete Question Explanation
(The complete setup for this game can be found here: lsat/viewtopic.php?f=177&p=88290#p88290)

The correct answer choice is (E).

Answer choice (A) can be eliminated because from the second rule T must be on the second-place team.

Answer choice (B) can be eliminated because from the third rule M and P cannot be on the same team.

Answer choice (C) can be eliminated because from the fourth rule P’s team places higher than N’s team.

Answer choice (D) can be eliminated because per the inference made in the discussion of the last rule, S can never be on the third-place team.

Answer choice (E) is the correct answer.

Get the most out of your LSAT Prep Plus subscription.

Analyze and track your performance with our Testing and Analytics Package.